You are on page 1of 88

www.byjusexamprep.

com

Mock Test Solutions in English

Questions

1. Direction: Read the passage below and answer the questions that follow.-

In Nineteen Eighty-Four, Orwell captures something fundamental to the use and abuse of power: its attack
on anything that smacks of rest. Whether it is within religious cults, army boot camps, torture chambers or
concentration camps, the exhausted self—shorn of the energy to do anything other than comply—is a
malleable self.

So does it follow that, as Tricia Hersey puts it in her new book, “rest is resistance”? Unprogrammed time
may be liberating.But there are limitations to rest as a form of subversion, too; all that Winston (in
Nineteen Eighty- Four) manages in the end is to carve out tiny moments of freedom before he is inevitably
crushed.

Hersey is an African-American writer, artist and activist. Her perspective is heavily influenced by black
feminist traditions, Afrofuturist thought and, above all, the Pentecostal church in which she was raised (her
father was an assistant pastor). Indeed, Rest is Resistance, a passionate polemic for the most part, reads
at times like a sermon or a prayer. The book draws on Hersey’s work as “bishop” of The Nap Ministry, the
project she founded. The Ministry explores the power of rest as a communal experience, including
“collective napping experiences” that turn everyday locations into “sacred nap spaces”.

Hersey proclaims in the book that “The more we rest the more we wake up”. She situates herself in
opposition to the nexus of white supremacy and capitalism (which are, for her in any case, virtually the
same thing). What she calls “grind culture” doesn’t just dominate our lives today; it is rooted in the
exploitation and exhaustion of the black bodies upon which, Hersey argues, America itself was founded.
Just as the slave’s body is not their own, so too does capitalism extend ownership over all our bodies,
albeit in more insidious and disguised ways. Naps and practices of rest reclaim our bodies and also our
stolen “dreamspace”, our capacity to imagine that things could be otherwise.

According to the passage, which of the following is the most likely reason behind the prayer-like quality of
Tricia Hersey’s book?
A. Hersey’s viewpoint has been shaped by black B. Hersey has African-American roots and has been
www.byjusexamprep.com

feminism. influenced by afrofuturism.


C. The book presents rest as a form of subversion. D. Hersey worked as a Bishop for The Nap Ministry.
E. Hersey was raised by an assistant pastor of the
Pentecostal church.
2. What does Hersey imply by the sentence ‘The more we rest, the more we wake up.’?
A. The more we relax, the more refreshed our body
B. The more we rest, the more empowered and
feels, which can help us in honing our creative
imaginative we become.
abilities.
C. The more we take naps or rest, the higher the D. The more we rest, the more we empower our
frequency of our waking up. body and imaginative abilities.
E. The more we rest, the more aware our body and
soul become.
3. All of the following can be concluded from the passage EXCEPT:
A. Rest does not help Winston much in the book B. There exists a similarity between white
Nineteen Eighty-Four. supremacy and capitalism.
C. An exhausted person is likely to be submissive in D. Hersey’s work emulates Nineteen Eighty- Four in
the face of authority. theme and style.
E. Resting is a freeing experience.
4. Direction: Read the poem carefully and answer the following TWO questions:

You fit into me

like a hook into an eye

a fish hook

an open eye

Which of the following best captures the theme of the poem?


B. It captures a complex relationship between two
A. It captures an intimate relationship between two
lovers that may be sometimes joyful and sometimes
lovers who are perfect for each other.
painful.
C. It captures a relationship that may appear happy D. It captures a relationship marked by physical
on the outside but is painful in reality. violence and abuse.
www.byjusexamprep.com

E. It captures a relationship where both the partners


are helpless in their relationship.
5. Consider the following statements:

1). The poem involves two parties.

2). The hook referred to in the second and the third line are likely to be the same.

3). The eye referred to in the second and the fourth line may not be the same.

Which of the given following statements are true?


A. 1, 2, and 3 B. Only 1 and 2
C. Only 2 and 3 D. Only 1 and 3
E. None of the above
6. Which of the following sentences are grammatically INCORRECT?

1). Given the history, the measures taken by the Board of Education are not surprising.

2). While private consumption had improved, consumer confidence is still pessimistic because of high
inflation.

3). Various symposiums on Jane Austen’s works are testimony for her cult following.

4). Calling all the consultants, apologise to them, accepting all their demands and offer them hefty hikes.

5). Elucidate the promotional measures used by car dealers to attract customers.

6). The word dharma has been misused from ancient times till now.
A. 2, 3, 4 B. 1, 2, 4
C. 1, 5, 6 D. 4, 5, 6
E. 3, 4, 5
7. Read the following paragraph carefully

Three themes ______ Churchland’s philosophical discussion: developing an _____ to the logical
empiricist theory of intertheoretic reduction; responding to property-dualistic arguments based on _______
and sensory qualia ; and responding to anti-reductionist multiple realizability arguments.
www.byjusexamprep.com

Which of the following options will BEST fill up the above blanks meaningfully?
A. neglected, idea, subjectivity B. approved, idea, unbiasedly
C. neglected, imperative, objects D. defying, alternative, expressions
E. predominated, alternative, subjectivity
8. Arrange the following sentences in a LOGICAL sequence.

1). Hence, the criticism of knowledge which philosophy employs must not be of this destructive kind if any
result is to be achieved.

2). For all refutation must begin with some piece of knowledge which the disputants share; from blank
doubt, no argument can begin.

3). If we adopt the attitude of the complete sceptic, placing ourselves wholly outside all knowledge, and
asking, from this outside position, to be compelled to return within the circle of knowledge, we are
demanding what is impossible, and our scepticism can never be refuted.

4). Against this absolute scepticism, no logical argument can be advanced.

5). When we speak of philosophy as a criticism of knowledge, it is necessary to impose a certain


limitation.
A. 3, 5, 2, 4, 1 B. 2, 3, 1, 4, 5
C. 5, 3, 2, 1, 4 D. 5, 1, 3, 2, 4
E. 2, 1, 4, 5, 3
9. Arrange the following sentences in a LOGICAL sequence.

1). Socrates points out that Simmias is taller than Socrates (and hence tall), but that Simmias is also
shorter than Phaedo (and hence short).

2). Hence, Socrates is both like and unlike.

3). Being like Plato, Socrates is like; being unlike Meletus, Socrates is unlike.

4). Clearly, Socrates envisages the possibility of sensible things being both like in one way and unlike in
another.
www.byjusexamprep.com

5). For example, Socrates is like Plato (in that each is a philosopher) and unlike Meletus (in that one, but
not the other, is a poet).
A. 1, 4, 5, 3, 2 B. 3, 5, 4, 2, 1
C. 3, 5, 1, 2, 4 D. 1, 2, 5, 3, 4
E. 3, 4, 2, 1, 5
10. Direction: Read the passage below and answer the questions that follow.

Last year, the Survey Center on American Life published a study tracking friendship patterns in the United
States. The report was anything but heartening. Registering a “friendship recession,” the report noted how
Americans were increasingly lonely and isolated: 12 percent of them now say they do not have close
friendships, compared to 3 percent in 1990, and almost 50 percent said they lost contact with friends
during the COVID-19 pandemic. The psychosomatic fallout was dire: heart disease, sleep disruptions,
increased risk of Alzheimer’s. The friendship recession has had potentially lethal effects.

The centre’s study offered a miniaturised model of a much broader process that has overtaken countries
beyond the United States in the last thirty years. As the quintessential voluntary association, friendship
circles stand in for other institutions in our collective life — unions, parties, clubs. In his memoirs, French
philosopher Jean-Claude Michéa said that one of the most disconcerting moments of his childhood was
the day he discovered that there were people in the village who were not members of the Communist
Party. “That seemed unimaginable,” he recalled, as if those people “lived outside of society.” Not
coincidentally, in May 1968, French students sometimes compared the relationship of workers to the
Communist Party with that of Christians to the church. The Christians yearned for God, and the workers
for revolution. Instead, “the Christians got the church, and the working class got the party.”

The son of communist parents, Michéa saw the party as an extension of a more primary social unit.
Friendship patterns have always served as a useful indicator for broader social trends, and political writers
were quick to apply the data to political analysis. The researchers invoked Hannah Arendt’s dictum that
friendship was the best antidote for authoritarianism. At the end of 1951’s The Origins of Totalitarianism,
Arendt postulated that a new form of loneliness had overtaken Westerners in the twentieth century,
leading them to join new secular cults to remedy their perdition. “What prepares men for totalitarian
domination in the non-totalitarian world,” she claimed, “is the fact that loneliness, once a borderline
experience usually suffered in certain marginal social conditions like old age, has become an everyday
experience.” The conclusions were clear. As Americans become lonelier and more isolated in the new
century, the same totalitarian temptation is likely to lurk.
www.byjusexamprep.com

Which of the following can be BEST concluded from the sentences ‘The Christians yearned for God, and
the workers for revolution. Instead, “the Christians got the church, and the working class got the party.”’
A. Each group ended up getting what the other B. The Church is a proxy for God, and the party is a
wanted. proxy for revolution.
C. The Communist Party and revolution and the D. Each group got an extension (church, Communist
church and God are not the same. party) of what it yearned for (God, revolution).
E. Each group got the opposite of what they longed
for.
11. Based on the passage, which of the following BEST describes the term ‘friendship recession’?
A. A fallout with friends leading to increased B. A decline in the number of friends as a person
loneliness and isolation grows older
D. A decline in the number of friends owing to the
C. A pattern of decrease in friendships
pandemic
E. An increase in loneliness and isolation in the
West
12. Which of the following, if true, would MOST undermine the argument made by the author towards the end
of the last paragraph?
B. There are many important factors besides
A. Loneliness, as described by Hannah Arendt, was
loneliness that prepare men for totalitarian
not a phenomenon unique to the twentieth century.
domination.
D. The conditions related to loneliness in the
C. Loneliness has always existed in our society, but
twentieth century are quite dissimilar to those of the
our society has not always been totalitarian.
current century.
E. A number of other countries have reported
increased loneliness, but none of them have given in
to the totalitarian temptation.
13. It is too early to tell whether the remarkable protests across China against the government’s strict
‘dynamic zero Covid’ policy are a milestone in the country’s long history that will have lasting impact; they
may signal growing discontent with the Chinese leadership or a more limited expression of popular
frustration. But it’s momentous all the same that thousands of Chinese people have raised their voices to
criticise their leaders’ policies in open view of police officers and the security apparatus, in the largest
mass demonstrations since the 1989 pro-democracy protests in Tiananmen Square.
www.byjusexamprep.com

For now, the protesters in China have made their voices heard under very difficult conditions. They
demonstrated the value of protest and dissent — freedoms that many in the world hold dear. At a time of
geopolitical hostility, they offer an opportunity to Americans and others to better understand the diversity of
views that exists within China and to see the Chinese people anew.

Which of the following cannot be inferred from the information given above?
A. The Chinese people’s protests against the B. The popular protests against the Chinese
government are unprecedented and unlikely to government are being viewed by the government as
succeed. an attempt at secession.
C. The present Chinese protests against the D. The bravery of the Chinese people is an
government is a case of history repeating itself in affirmation that freedom of expression, dissent, and
China. protest are held by people all over the world.
E. The world should support the Chinese protestors
to overthrow their government.
14. Carefully read the following paragraph.

The antiquity and sheer durability of ________ civilisation gave the ancient Egyptians a ________ sense
of their own superiority. They _______ the confidence that they—and they alone—were the _________ of
a universe brought into being at the time of creation, a world unchanged and unchanging.

Fill in the blanks with appropriate options.


A. pharaoh, hoarding, surged, forefathers B. pharaoh, ambiguous, dribbled, forefathers
C. pharaonic, profound, exuded, inheritors D. latest, feeble, transude, inheritors
E. infinitesimal, dubious, surged, ancestors
15. Thinking of human agency as an individual and independent activity is misguided. At best, individualism
leads to clumsy and ineffective courses of action and, at worst, to conflict, chaos, and harm. Yet nothing in
this theory of co-action intends to take power and control away from humans. Counterintuitively, it’s in the
act of acknowledging that we aren’t in full control of our actions, that we gain more control over their
outcomes, and greater imaginative scope to create more harmonious and effective social structures. In
order to more justly attribute responsibility, guilt and merit across the field of entangled actors, and to
design social systems that offer more equitable and nurturing paths for the future, we must look to the
others around us who participate in our agency. Casting human agency in a more distributive and
www.byjusexamprep.com

collective mode will help us to become more genuinely autonomous—as interwoven, interdependent, and
relational beings.

Which of the following statements is best concluded from the paragraph given above?
A. Human beings are interwoven and interdependent
B. We are misguided when we work independently
and can become more autonomous with collective
and individually.
action.
C. One can gain control over oneself and others by
D. Individuals do not participate in societal activities
taking a different perception of power and how it
as society is not designed equitably.
operates.
E. Theories that are counterintuitive are not always
incorrect and rationality is not always correct.
16. Carefully read the following paragraph.

You see, he was of a/an _______ disposition; therefore, it took him _____time to get tired of arguing with a
person who agreed with everything he said. He retreated and sat in a corner with a look of ______ on his
face.

Fill in the blanks with appropriate options.


A. amiable, a little, satisfaction B. agreeable, some, dread
C. argumentative, little, boredom D. joyful, little, glee
E. quarrelsome, a long, victory
17. Direction: Read the passage below and answer the questions that follow.

To compare the coffeehouse tradition to the social media networks of today may seem far-fetched. But
consider the parallels. Across its various manifestations, the coffeehouse typically fused news and debate
in one space. People would go there to find out what was going on, discuss and take a view on it.

Coffeehouses specialised in certain fields of interest or trade: lawyers, printers, merchants and insurers all
had their favoured locales in 18th-century London. Some even developed into private exchanges and
clubs; the London Stock Exchange and Lloyd’s of London both began as coffeehouses. But most were
fundamentally open and democratic, albeit male-centric, allowing in any man who could pay the price of a
coffee. These institutions generated their own formats and cultural peculiarities: pamphlets, newspapers,
journals, stock indexes, newsletters and aphorisms.
www.byjusexamprep.com

The American and French revolutions had roots in coffeehouses: the former at the Merchants’ Coffee
House in New York, where resentments at British rule swirled in the years running up to 1775; the latter at
the Café de Foy in Paris, where the revolutionary lawyer Camille Desmoulins fired up the patrons to march
on the Bastille in 1789.

Debates raged about the merits and demerits of coffeehouses. Their fans argued that they sharpened
wits, stimulated debate and democratised information. Critics deemed them time-wasting, seditious,
boastful, unmanly, mob-minded and intellectually unserious. The satirist Jonathan Swift warned against
mistaking “the echo of a London coffeehouse for the voice of the kingdom”.

Much of this applies to social media today. Twitter, with its slogan “It’s what’s happening”, markets itself as
a space where news is not just shared, but also discussed and made. It displays a remarkable openness
whereby anyone can engage directly with anyone else. It has generated new forms of expression:
threads, hashtags, the short viral video clip, “moments” and memes (the aphorisms of our time). Platforms
such as Twitter and Facebook have helped campaigners and revolutionaries spread their messages to
others – from the Arab Spring to the Black Lives Matter movement.

Debates rage over the role of social media networks, just as they once did over that of the coffeehouse.
They stand accused of stoking precisely the same social ills. Consider “The Character of Coffee and
Coffeehouses”, written by John Starkey, in London in 1661. His pamphlet can be read as both an account
of its time and as an uncannily apt commentary on Twitter and the like today.

Which of the following BEST sums up the purpose of the author behind writing the passage?
A. Highlight the history and similar debates over B. Draw an analogy between traditional coffee
coffee houses and social media. houses and social media to clarify a concept.
C. Compare and contrast the merits and demerits of
D. Show that networking in traditional coffee houses
conversations in traditional coffee houses and social
and social media is similar in certain respects.
media posts..
E. Argue that an online space has replaced a
traditional space because of their similarities.
18. Which of the following can be reasonably inferred from the passage?

I. Coffeehouses promoted mob-mentality and instigated revolutions.

II. Social media platforms can stoke disturbances in the society.


www.byjusexamprep.com

III. Coffeehouses are similar to social media platforms such as Facebook and Twitter.
A. Only I and II B. Only III
C. Either I or II D. None of the statements can be inferred.
E. All the statements can be inferred.
19. What is the purpose of the author behind mentioning ‘private exchanges and clubs’ in the passage?
A. To show that coffeehouses functioned as close- B. To emphasise the democratic nature of
knit groups. coffeehouses.
C. To illustrate the niche nature of coffeehouses, D. To give examples of private exchanges and clubs
which led to their becoming exchanges and clubs. that began as coffeehouses.
E. To exemplify the evolution of some coffeehouses
owing to their niche nature.
20. Direction: Read the passage below and answer the questions that follow.

There is an implicit argument that wisdom is something separate from the question of group identity. As
the psychologist Igor Grossmann noted in 2020 in his helpful summary of the state of wisdom studies, one
of the key goals is to find a definition of wisdom that transcends individual cultures by identifying common
threads around the world. Once we look at the emerging definition more closely, however, we may notice
something interesting.

The two key features of the current definition are: moral groundedness and metacognition. Grossmann
writes that the first implies ‘ideas such as the sense of shared humanity, pursuit of truth, recognition of the
need to balance personal interests with other people’s interests, and a general willingness to cooperate
and have compassion for others.’ This is very similar to what we find in the humanist definitions.

Metacognition, meanwhile, is about your ability to reflect on your own thinking: showing humility, listening
to others, learning from your experiences. While Grossmann does not draw a direct line between this
second feature of wisdom and engaging cultural differences, there is an obvious parallel. To be wise does
not only mean learning about what we share as humans. It also means being able to work through our
differences reasonably without abandoning our moral insights.

Perhaps surprisingly, this challenge to unite shared humanity and cultural identity is not unique to the
modern world. If we look at some of the classic texts to which these writers refer – Greek philosophy and
the Hebrew scriptures – we find continual reflections on the relationship between cultural particularism and
universal wisdom. While the quest for a common humanity is certainly present in ancient texts, there is
www.byjusexamprep.com

also an idea in wisdom traditions that, to truly be wise, we have to learn to live with different ideas of the
truth. Wisdom in this sense is not separate from identity, but is about figuring out how humans can live well
and develop universal standards of wellbeing, while also knowing that no single standard of wellness will
ever be meaningful to everyone. And, given that our cultures and communities do not fully define us, this
is as true within cultures as between them.

Which of the following expressions captures the passage’s concern MOST comprehensively?
A. Unity in diversity B. United we stand, divided we fall
C. To err is human, to forgive divine D. I am wise because I know that I do not know
E. We live in everyone
21. According to the passage, which of the following can be said to be true?
A. Cultural differences present a unique challenge to
B. Wisdom is definable in words.
attaining wisdom today.
C. Moral groundedness and metacognition are
D. Wisdom is separate from group identity.
indispensable to attain wisdom.
E. Wisdom is universal like the laws of nature.
22. Whom is the author LEAST likely to support to acquire wisdom?
A. Ms. X working on her empathy and listening skills B. Mr. Y studying Greek philosophy and Hebrew
so that she can cooperate with her fellow human scriptures and concluding that wisdom has a dual
beings. nature.
C. Ms. Z studying the arguments on moral D. Mr. A working to find a new definition of wisdom
groundedness in wisdom studies for a better that incorporates shared humanity and cultural
understanding of wisdom. diversity.
E. Ms. B closely analysing the current definition of
wisdom and concluding that it is applicable intra-
culturally and inter-culturally.
23. Few topics annoy green activists as much as air travel. Planes account for a sliver of total carbon-dioxide
emissions—roughly 2.5%—but unlike heating, other transport, and electricity, journeys on them are often
luxuries not essentials. Air travel has also largely escaped carbon pricing which covers a quarter of all
emissions.

Change is in the air, however. Since the start of last year, a UN-backed scheme has required airlines to
offset emissions above a baseline by buying credits. The EU has decided to bring airlines deeper into its
www.byjusexamprep.com

carbon-trading scheme. These are steps in the right direction, even if they are unlikely to bring about fast
enough decarbonisation for Europe to hit its goal of a net-zero aviation industry by 2050.

Which of the following can BEST be concluded from the above passage?
A. Air travel has escaped carbon pricing as its
B. The EU has dragged its feet on bringing airlines
emissions are negligible compared to other forms of
into its carbon trading plan.
travel.
D. The EU needs to implement its schemes
C. The EU might be able to meet its net zero
immediately to meet its net zero aviation target by
aviation emissions after 2050
2050
E. The airlines have escaped carbon pricing as air
travel cannot be avoided in modern life.
24. Camouflage is the by-product of an evolutionary arms race between one group—the predators—that want
to eat another group—the prey.The prey seek to survive by tricking the perceptual and cognitive systems
of the predators. The predators look to eat by breaking through the camouflage of the prey.

To put it all more formally, predators need to maximise their perception of signal (their proposed meal)
from the noise (the general environment); the goal of prey is to minimise that same signal-to-noise ratio.
While the physical manifestations of camouflage might be diverse, they function in one of three ways: to
impair detection, to impair identification, or to redirect attack.

Which of the following is not an assumption implicit in the argument?


A. There is a complicated evolutionary conflict
B. Disguise is a tool of survival for the prey.
between predator and prey.
C. Camouflage works in nature and the natural D. The manifestations and working of camouflage in
world. nature has limitations.
E. Predators and prey are aware of their mutual
survival strategies.
25. Humans have been honed over millions of years of evolution to respond to certain situations without
thinking too hard. If your ancestors spotted movement in the undergrowth, they would run first and grunt
questions later. At the same time, the capacity to analyse and to plan is part of what distinguishes people
from other animals. The question of when to trust your gut and when to test your assumptions—whether to
www.byjusexamprep.com

think fast or slow, in the language of Daniel Kahneman, a psychologist—matters in the office as much as
in the savannah.

Deliberative thinking is the hallmark of a well-managed workplace. Strategic overhauls and budget
discussions are built on rounds of meetings, memos, formulas, and presentations. Processes are
increasingly designed to stamp out instinctive responses. From blind screening of job applicants to using
‘red-teaming’ techniques to pick apart a firm’s plans, rigour trumps reflex.

Yet instinct also has its place. Some decisions are more connected to emotional responses and inherently
less tractable to analysis. Does a marketing campaign capture the essence of your company, say, or
would this person work well with other people in a team? In sticky customer-service situations, intuition is
often a better guide to how to behave than a script.

Which of the following conclusions follows from the passage?


A. Intuition is an inherited trait which is not very B. Intuition is crucial for survival both in the office
common among managers. and in nature.
C. Managers are trained to use deliberative thinking D. Instinctive thinking is superior to deliberative
to the exclusion of instinctive thinking. thinking in most office situations.
E. In some situations, a manager is required to trust
his instinct.
26. Which of the following sentences are grammatically INCORRECT?

1). The scientists used a chainsaw to cut several cross-sections from the vessel’s planking.

2). The developed nations are now taking action to make more carbon space available for others.

3). It starts with listening to diverse point of views.

4). Just setting the standard is useless, yet the laws are not implemented religiously.

5). The globe needs radical change, but nations tend to have narrower perspectives.

6). We can say that domestic work are not counted under labour legislation.
A. 3, 4, 5 B. 1, 2, 3
C. 3, 4, 6 D. 4, 5, 6
www.byjusexamprep.com

E. 2, 3, 4
27. Raveena, who is working at a leading tech company, also runs a food van near her home on the
weekends. She sells chicken sandwiches and veg wraps from her food van. The food van is a favourite
among college students who form a major part of her clientele as the food offered is pocket friendly (Rs.
60–70). The students visit the van generally for evening snacks and prefer the sandwiches and wraps sold
by Raveena over the ones offered by leading food chains. She is able to sell 550–650 units on the
weekend.

One of Raveena’s customers, Akshay, asks her to open a stall in the college he is studying in. He offers to
run the stall on the weekdays if she pays him a minimal amount each month.

Which of the following will persuade Raveena to open a stall in the college?
A. The college authorities charge Rs. 50,000 from B. There is no food stall selling chicken sandwiches
each food stall irrespective of the sales. in the college.
C. The college authorities ask for a non-refundable
D. The costliest dish in the food court is priced at Rs.
deposit of Rs. 5 lakhs from people setting up any
55
new food stalls in the college.
E. Students in the college generally eat from their
favourite places.
28. Raveena sets up a stall at the college which is run by Akshay. The stall sells a chicken sandwich for Rs.
65 and a veg wrap for Rs. 55. The stall is operational from Monday to Saturday and sells only 400 units in
the first month of operation. Raveena is not convinced with the sales numbers and doubts that Akshay is
not reporting the correct figures to her. Which of the following steps should Raveena take to address the
issue?

I. File a police complaint against Akshay.

II. Ask some of the other college students to keep an eye on Akshay.

III. Discuss the issue with Akshay and ask him for an explanation.

IV. Install CCTV cameras near the food stall in college.

V. Visit the food stall randomly without informing Akshay.


A. I, II, and III B. II and V
www.byjusexamprep.com

C. I and II D. III, IV, and V


E. IV, II, and I
29. Raveena is not doing well in her job while the food stall turns profitable in three months. Akshay keeps
coming up with new ideas to expand the business. However, Raveena also realises that Akshay’s final
exams are approaching and he won't be available for approximately 20 days next month. What should
Raveena do?

Consider the following courses of action.

I. Ask Akshay to leave now as he won’t be available next month.

II. Offer a bonus to Akshay if he turns up for work for more than 20 days next month.

III. Talk to the Principal of the college to postpone the exams as Raveena might lose her job.

Select the best course of action for Raveena.


A. I only B. II only
C. III only D. II and III
E. None of the above
30. Direction: Read the caselet given below and answer the questions that follow.

Jeewan Hospital is a private hospital in the village of Bhadohi in UP. It has been the best hospital in the
whole district since its inception in 1975. The hospital always advertised its infrastructure, latest
technologies, well-trained staff, and availability of oxygen and ventilators. The hospital also boasts of a
general ward with a capacity of 25 patients where it provides free medical services to the needy. You
recently visited the hospital and noticed that the security guards did not let poor people even enter the
premises of the hospital irrespective of their medical condition.

While you were at the hospital, you saw a young man begging for mercy as he wanted to admit his
pregnant wife who was experiencing some complications. The security guard refused to let them inside
the hospital saying that there were no beds available in the general ward. However, you found out that the
security guard was lying.

Which of the following combinations of the given actions will help the young man admit his wife to Jeewan
Hospital the most?
www.byjusexamprep.com

1). Reach out to the Chief Medical Officer at the hospital and explain the situation to them.

2). Write an email and tweet to the Health Minister of UP.

3). Take some pictures of negligence at the hospital and threaten the authorities with dire consequences.

4). Approach the doctors on duty in the emergency department and explain the situation to them.

5). Talk to the people at the reception and tell them about the situation.
A. 5, 2, 3 B. 5, 4, 1
C. 2, 1, 4 D. 2, 3, 4
E. 3, 4, 1
31. Which of the following would best help the management of Jeewan Hospital to improve their image in the
eyes of the needy?
A. Set up beds for the needy on the footpath outside B. Set up health camps on a first-come-first-serve
the hospital. basis in the hospital premises.
C. Provide a 10% discount across all services to the D. Provide free OPD services and admission to the
needy people. needy.
E. Provide free fruits and fruit juices to children.
32. When you came back from Bhadohi, you wrote a blog on the state of medical services in Uttar Pradesh
and mentioned Jeewan Hospital as an example of non-inclusivity. Your blog has already been read by
10,000 people and has reached some of the eminent personalities in the social sector. These people have
also reached out to the management of Jeewan Hospital. You then received a call from the PR team of
Jeewan Hospital narrating the whole story to you and asking you to add good things about them in the
comments to the blog or take down your blog completely.

Given the circumstances, what should be the ideal course of action for you?
A. Stick to the original comments as you have B. Add some nice pictures of Jeewan Hospital to
reported exactly what took place. change people’s opinion.
C. Delete this blog and replace it with a new one D. Edit the same blog, especially the sections on
praising Jeewan Hospital. Jeewan Hospital.
E. Ask the PR team for monetary compensation in
order to make the changes they want.
33. Direction: Read the caselet given below and answer the questions that follow.
www.byjusexamprep.com

Rupiya is a cooperative bank which was established in 1975. It has six branches in Delhi and employs 60
people. The bank always functioned smoothly with the help of the board of directors. The bank always
valued experience and retained employees with more than five years of experience even during bad
times. The bank has a reputation for being employee-friendly and helping its members during bad times
by providing various kinds of loans to them.

Recently, Mr. Suraj, who is the son of one of the members of the board of directors, joined the bank as the
Managing Director. He is an MBA graduate in finance from a premier B-school and wanted young talent to
take the bank to new heights. He therefore hired six fresh MBA graduates from leading B-schools and
placed the six MBA graduates as the branch managers for six respective branches.

Mr. Suraj wanted to take all the data to the cloud which required the skill and expertise of the older
members. He asked the older members to assist the new joinees in doing so without asking them any
questions. The old timers agreed to his demands but they started coming in late to the office and the
absenteeism went up too. Also, during the board meeting, the board members kept quiet about Mr. Suraj’s
decisions and complaints about Mr. Suraj’s decisions started coming in from various branches.

Which of the following seems to be the most valid reason for coming late and absenteeism of the old
timers?
B. Six fresh MBA graduates being appointed as
A. Mr. Suraj being appointed the Managing Director
branch managers
C. New talent and old-timers not getting along D. Single-handed decision-making by Mr. Suraj
E. Incompetence of Mr. Suraj
34. How should Mr. Suraj handle the situation now? Choose the best option.
B. Promise perks and salary hikes to the new
A. Go on a leave for 10 days and let things settle on members and ask them to form alliances with the old
their own. members so that they don’t get into the habit of
coming late and absenteeism.
C. Reach out to people not coming to work and
D. Resign from the bank and issue an apology to the
request them to come back and promise them perks
old-timers, asking them to come back to work.
and salary hikes.
E. Reach out to the people coming in late or not
coming at all, discuss the matter with them, and
www.byjusexamprep.com

create joint teams of old and new members.


35. Which of the following would have been the best combination of actions, in the correct order, in order to
move the data to the cloud by Mr. Suraj?

1). Fire all the old timers and hire fresh talent.

2). Ask employees at the various branches of the bank to share their suggestions.

3). Call for a board meeting and seek ideas from the board members.

4). Take a collective decision with the board of directors.

5). Ask the old-timers to take the decision on his behalf and take a backseat himself.
A. 2, 3, 4 B. 3, 4, 5
C. 1, 3, 4 D. 2, 5, 4
E. 3, 4, 1
36. Direction: Read the caselet given below and answer the questions that follow.

You recently joined the Human Resource (HR) department of XYZ Consulting as an HR manager
immediately after you finished your MBA in HR from a leading B-School. You assist your manager, Ms.
Shelly Rhodes, in conducting training sessions and conducting employee engagement programmes.

While studying the data and exit forms of various employees in the past five years, you came across the
record of an employee, Mr. Rahul. He resigned from the company in 2018 but neither there is a reason for
his exit mentioned in the database nor does his exit form exist anywhere in the records.

Which of the following is the best course of action for you?


B. Report the issue to the head of the department
A. Report the issue to the HR head of your company.
Mr. Rahul belonged to.
C. Look for Mr. Rahul’s contact information on D. Bring the issue to your manager’s notice and seek
Linkedin and reach out to him for clarification. their advice.
E. Reach out to people who were present in the
company in 2018 and interview them.
37. You overheard a conversation between two employees. They were talking about the exit of Mr. Nayyar two
months ago, who was asked by his manager to resign as he was accused of sexual harassment by Ms.
www.byjusexamprep.com

Rubina in his team. On further investigating the issue, you found out that this issue was never reported to
the HR department or the sexual harassment prevention committee as the manager felt that Mr. Nayyar
was innocent and reporting to the HR could ruin his career. Also, Ms. Rubina is still working in the
organisation while Mr. Nayyar is still out of work.

Had you been the director, what would be the first thing you would have done?
A. Ask Mr. Nayyar to stop coming to office and B. Counsel Ms. Rubina to keep mum about the
resign. whole issue.
C. Report the matter to the sexual harassment D. Take help of some senior to conduct a thorough
prevention committee. investigation.
E. Change the departments of both Mr. Nayyar and
Ms. Rubina.
38. Ms. Shelly Rhodes makes you in charge of the sexual harassment prevention committee. Ms. Nupur
reaches out to you complaining about her manager, Mr. Anshul. According to Ms. Nupur, Mr. Anshul has
been dating her for the past two years on the pretext of marrying her but now Mr. Anshul has started going
around with the new hire from Canada, Ms. Wolowitz. Ms. Nupur also cited some personal chats and
phone calls to support her case. Now she wants the sexual harassment prevention committee to take
strict action against Mr. Anshul.

Which of the following actions should you take?

1). Issue show cause notices to Mr. Anshul and Ms. Wolowitz.

2). Issue show cause notices to Mr. Anshul.

3). Issue show cause notices to Ms. Nupur.

4). Discuss the whole matter with the sexual harassment team and come up with a plan of action.

5). Ask Mr. Anshul to leave office with immediate effect to save him from embarrassment.
A. 4, 1, 5 B. 3, 1, 5
C. 3, 2, 1 D. 4, 3, 1
E. 4, 3, 5
39. Ceenu is a team lead in an ed tech company called Jujubes. He looks after the content and training
department at the company. Apart from managing a team of 10 people in the content department and 8
www.byjusexamprep.com

people in the training department, he also takes training sessions whenever required. Sachin, who is the
product manager, has told Anju to schedule sessions for Ceenu only when no one else is available. Anju
takes care of scheduling at Jujubes.

Ceenu had to play in a cricket tournament on a particular weekend. So he informed Anju two weeks in
advance about him not being available on that weekend. Anju did not make a note of this and scheduled a
session to be conducted by Ceenu on the day of the tournament. He was informed about the workshop a
day before the tournament. He reminded Anju about his non-availability and asked her to arrange for
another faculty member. Anju could not arrange for another faculty member and, therefore, escalated the
whole issue to Sachin who in turn holds Ceenu accountable for the confusion.

Consider the following courses of action:

I. Ask Anju to send an apology mail to him marking the whole team if he wants him to conduct the session.

II. Get the session timings adjusted so that the timings do not clash with the tournament and take the
session.

III. Call Sachin and tell him that Anju is fond of causing inconvenience to him.

IV. Have a meeting with Sachin and Anju to clarify everything.

Mark the option which includes the right things for Ceenu to do.
A. I and II B. I and IV
C. II and III D. II and IV
E. I and III
40. Ceenu became cautious after this incident and stopped conveying any important messages verbally. He
started writing emails for all the important communication. Anju also became used to this and the working
relationship between Ceenu and Anju improved. Anju took a sabbatical for three months and transferred
all her responsibilities to Tamanna. Ceenu became extra cautious while dealing with Tamanna and
followed the same practice of conveying important messages via emails. One day, Ceenu sends an email
to Tamanna to schedule a session for him with a group of students who had complained to him regarding
the bad behaviour of a few faculty members in their classes. The session was to be scheduled on a
particular date as advised by Sachin. Two days before the session, Sachin reprimanded Ceenu for not
getting the session scheduled as informed by a few students. Ceenu told him that he communicated about
www.byjusexamprep.com

the session to Tamanna two weeks ago and it is indeed scheduled. When Ceenu asked Tamanna about
the session, she told him that she never received any email regarding the session. Ceenu could not
believe Tamanna because of repeated complaints about Tamanna not getting emails.

What should Ceenu do?


A. Take Sachin into confidence and get the session
B. Call Anju to help him this time.
scheduled.
D. Call Tamanna and tell her that he can get her fired
C. Do not do anything as nothing can be done now.
if she does not handle the situation.
E. Connect with students with his personal account
and conduct the session.
41. Anju resumed work after a three-month sabbatical and Tamanna was transferred to another role wherein
she was supposed to take care of the app for the product. She was supposed to make sure that
everything on the app is up to date. A student reaches out to Anju with the screenshot of the app wherein
mock test 8 was not live even after two days of the scheduled date. Anju reaches out to Tamanna to get
the issue resolved. In order to escape working, Tammana told Anju ‘I do not work on this. Reach out to
someone else.’ Anju, who is already overburdened, finds it difficult to identify any other person to take care
of the issue. The issue remains unresolved.

Consider the following:

I. Wait for the student to escalate the issue further and Tamanna to work.

II. Write a formal email to Tamanna, marking Sachin, and persuade her to get the issue resolved.

III. Reach out to Sachin to understand Tamanna’s roles and responsibilities.

IV. Share Tamanna’s email ID with the student and ask them to write a stern email to Tamanna.

Which of the options would help Anju to resolve the issue?


A. I only B. III only
C. II and III D. II, III and IV
E. All of the above
42. Direction: Read the caselet given below and answer the questions that follow.
www.byjusexamprep.com

Ramesh, a middle-aged company executive, suddenly developed chest pains one morning at the office.
His colleagues promptly took him to RMNI hospital nearby for treatment. The doctor in charge conducted
various tests on Ramesh and came to the conclusion that there was nothing wrong with Ramesh except
some uneasiness caused by eating a lot of heavy food. The doctor prescribed some medication for this
ailment and discharged Ramesh. While Ramesh was walking back to his car in the hospital parking lot, he
suddenly collapsed. He was promptly rushed to the emergency section of the hospital where doctors tried
for 45 minutes to resuscitate his heart to no avail. Ramesh had passed away. Ramesh’s chagrined
colleagues and family members (who had quickly arrived) accused the doctor of gross negligence. But the
doctor was adamant that he was correct in initially discharging Ramesh since the tests conducted had
shown no sign of an impending heart attack. Ramesh’s family then took to social media to air their
grievances. This quickly drew over 15000 comments, almost all negative. The management acted by
terminating the doctor’s services and posting this action on social media. This failed to assuage most
people and drew a flurry of more negative comments from more people.

Who/What is responsible for the situation becoming unmanageable?

I. The hospital management for terminating the doctor’s services based on social media reactions.

II. Ramesh and his colleagues for having chosen RMNI hospital in the first place.

III. The doctor for failing to detect the impending heart attack.

IV. The hospital management for posting the doctor’s dismissal on social media.

V. The doctor for not conducting appropriate tests on Ramesh.


A. I and II only B. I, III, and IV only
C. I, III, and V only D. I and IV only
E. II and IV only
43. Soon the flurry of negative comments about RMNI hospital began to spiral out of control. Footfall at the
hospital began to fall sharply. Most of the negative comments pertained to the hospital publicising the
termination of the doctor. Around the same time, the apex body for cardiology, ICA, made a statement that
certain types of impending heart attacks are virtually undetectable. This news was widely disseminated in
the media.

Which of the following is the best way forward for the hospital at this juncture?
www.byjusexamprep.com

A. Unconditionally reinstate the doctor to his former B. Apologise to and reinstate the doctor and post this
position and publicise this on social media. news on social media.
C. Try and find out as to whether the type of heart
D. Recruit another doctor and stick to the original
attack that Ramesh suffered from was undetectable
decision of firing the doctor.
and then make a decision.
E. Refrain from posting on social media with
immediate effect and delete all social media
accounts.
44. As the social media posts surrounding the circumstances behind Ramesh’s death refused to die down,
some eminent medical professionals posted the news that RMNI hospital was not a specialty heart
hospital and the hospital should ideally have referred Ramesh to another hospital.

If the posts made by the medical professionals are true, what should the doctor ideally have done?
B. After the tests, the doctor should have
A. The doctor should have immediately advised immediately consulted cardiologists over phone,
Ramesh to consult another hospital. zoom meetings, or video-conferencing to find the
best course of action for Ramesh.
C. The doctor should have consulted his superiors D. After the tests, the doctor should have advised
after the tests to find the best course of action for Ramesh to consult an appropriate hospital for further
Ramesh. tests.
E. The doctor should have posted on social media
that he was duty bound by hospital rules not to
recommend Ramesh to another hospital.
45. Direction: Read the caselet given below and answer the questions that follow.

India is the world’s largest producer of oilseeds but the demand is so high that the country still has to
import oilseeds. Despite the setting up of an autonomous Oilseeds Board in 2005, the country is yet to
gain self-sufficiency in this valuable cash crop. Oilseeds are grains used for processing cooking oils such
as groundnut, coconut, mustard, sesame, soyabean, and sunflower. These grains are rich in protein and
nutritional value and the government would want all cooking oils to be processed from oilseeds only. The
Indian government is worried and may hold top officials of the Board responsible if it fails to meet
production targets and punish them with fines and transfers. The strict performance parameters are aimed
at ensuring that at least the reduced target of 40 million tons of oilseeds production per year is achieved.
www.byjusexamprep.com

While the target for the 13th five year plan (2017–2022) has already been scaled down from 48 million
tons per year to 40 million tons, only an average of 30 million tons per year was achieved in the first four
years of the plan.

If you were the Chairman of the Oilseeds Commission, which of the following statements (all of which are
assumed to be true) could be used to strengthen your case against the government holding top officials
responsible?

I. Due to drought, fresh planting of oilseed crops in many areas of the country was not possible between
2017–2021

II. The actions of the mid-level officials in the Commission are not in line with the objectives laid down by
the top management.

III. The delays have been due to difficulties in obtaining loans. The loans were delayed due to the funds
being diverted to various infrastructure projects.

IV. We are not to blame. The government is doing nothing to encourage multiple cropping in areas that still
have monocropping.

V. The reduced target is also unrealistic since land under rain-fed farming is almost saturated and the
government is not doing enough to encourage irrigated farming.
A. I, II, III, and IV B. I and III
C. II and IV D. I, II, and III
E. I, III, and IV
46. If you were a government representative, what would be your statement to motivate the Oilseeds Board to
work better?
A. Make a statement like ‘Nothing is impossible. B. Remind the board members that the country may
With hard work and dedication, any task can be have to import oilseeds from its scarce foreign
accomplished.’. exchange reserves if targets are not met.
C. Remind the board members that cooking oils
processed from oilseeds are healthy and nutritious D. Offer financial incentives and promotions to Board
for people consuming them. All other alternatives are members if targets are fulfilled.
relatively unhealthy.
www.byjusexamprep.com

E. Make a statement expressing shame to the Board


members for even the reduced targets not being
achieved.
47. Suppose the government further reduces the target for oilseeds production to 28 million tons per year for
the fifth year. Will the new target be met?
A. Yes B. Cannot be determined
D. Depends on the quantity of rainfall during the
C. No
sowing season
E. None of the above
48. What is the remainder when is divided by (x – 1)(x – 2)?
A. 6x – 5 B. 5x – 6
C. 3x – 4 D. 4x – 3
E. 7x - 1
49. If , find the value of

A. 343 B. 332
C. 322 D. 312
E. 310
50. A tangent PA touches the circle at A and AB is the diameter of the circle. The circle bisects the line PB.
Find the area of the triangle (in sq units) PAB if the radius of the circle is 4 units.
A. 16 B. 32
C. 64 D. 128
E. 256
51. 216log 64 = 10x + 2log y + 62 and log (24 + log x) = 3. Find the value of y – x.
2 3 3 5

A. 243 B. 118
C. 125 D. 378
E. 250
52. Find the number of integral values of x such that that satisfies the given inequality.

(3 – x)(5 – x)(7 – x)…………………..(91 – x)(93 – x) 0


A. 64 B. 66
www.byjusexamprep.com

C. 67 D. 68
E. 69
53. Out of all the two-digit and three-digit integers between 1 to 150, a number has to be selected at random.
What is the probability that the selected number is a two-digit number but not divisible by 9?
A. B.

C. D.
E. None of the above
54. Amar, Akbar, and Antony work in the same department. Amar takes four times the number of hours
required by all three of them to complete the work. Akbar takes 2 hours more than the number of hours
required by all three of them to complete the work. Antony takes 4 hours more than Akbar to complete the
work. If the payment received by three of them is Rs. 2400, find the money received by Akbar.
A. Rs. 800 B. Rs. 1000
C. Rs. 1200 D. Rs. 1600
E. None of these
55. What is the largest number that can always divide ?
A. 17 B. 2
C. 34 D. 51
E. None of the above
56. Ayush takes 16 hours to travel a distance of 144 km. He covers some part of his journey by bicycle and
some part by bike. The speed of the bicycle is 5 km/h and the speed of the bike is 13 km/h. Find the
distance (in km) covered by the person on the bicycle.
A. 50 B. 60
C. 80 D. 40
E. 30
57. Direction: Read the following passage and answer the questions.

Seerum Institute of India Pvt. Ltd., which is one of the pioneering vaccine manufacturers in the world,
developed a vaccine for Beepox. After having administered a total of 1 million doses during a tenure of 12
months, there were some symptoms that were associated among the recipients. A total of 430 such
recipients were surveyed, and the issues were noted.
www.byjusexamprep.com

Each such recipient’s issue can be categorised as one among the 12 medication errors that have been
listed in the primary(left) horizontal axis. The Pareto Chart given below follows the 80 : 20 rule and
discusses such errors among the 430 recipients.

A maximum of how many of the mentioned medication errors constitute exactly 20% of the total surveyed
recipients?
www.byjusexamprep.com

A. 6 B. 2
C. 3 D. 4
E. 5
58. A minimum of how many of the mentioned medication errors constitute exactly 50% of the total surveyed
recipients?
A. 4 B. 3
C. 5 D. 6
E. 7
59. In how many ways the medication errors can constitute exactly 20% of the total surveyed recipients?
A. 7 B. 8
C. 5 D. 4
E. 6
60. The total number of ways in which n different items can be distributed in four groups of 2, 2, 3, and x is
34650. Find the value of n + x.
A. 12 B. 11
C. 15 D. 13
E. 14
61. In a college fest, the maximum duration of a presentation is 12 minutes and the minimum duration is 6
minutes. The average duration of the presentation is 10 minutes. 4 presentations took 10 minutes or more
than 10 minutes each. If the total duration is 3 hours, find the maximum number of presentations
performed for exactly 10 minutes.
A. 12 B. 13
C. 14 D. 15
E. 16
62. Directions: The question given below is followed by two statements, I and II. You have to determine
whether the data given in the statements are sufficient for answering the question. You should use the
data and your knowledge of mathematics to choose the best possible answer.

In the figure given below, PQR is a triangle. PS and QT are two of the medians. What is the area of the
triangle QTR?
www.byjusexamprep.com

Statement I: PG : GT = 2 : 1

Statement II: PQ = 20 cm and QR = 40 cm


A. The data in statement I alone is sufficient to B. The data in statement II alone is sufficient to
answer the question, while the data in statement II answer the question, while the data in statement I
alone is not sufficient to answer the question. alone is not sufficient to answer the question.
C. The data either in statement I or statement II D. The data in both statements, I and II, together are
alone is sufficient to answer the question. necessary to answer the question.
E. The data in both statements, I and II, together are
not sufficient to answer the question.
63. Find the maximum value of n in so that always divides 181!.

A. 22 B. 33
C. 44 D. 55
E. 66
64. Direction: Read the following passage and answer the questions.

In the Marvel Sports Club, one-third of the members who like to play billiards also like to play snooker. The
number of members who like to play only snooker is one-fourth of the number of people who like to play
only squash. The number of members who like to play these three games is equal to the number of
www.byjusexamprep.com

people who do not like to play any of the three games. The number of people who like to play only billiards
and squash is the same as the number of members who like to play only squash and snooker, which is
equal to half the number of members who like to play billiards and snooker. The number of members who
like to play only billiards and snooker and the number of members who like to play only snooker are equal,
and each of them is equal to twice the number of members who do not like to play any of the three games.

If the number of members who like to play squash is 48, then the number of members who like to play
only billiards is:
A. 16 B. 18
C. 24 D. 21
E. 12
65. If the total number of members in the Marvel club is 430, then how many members like to play all the three
games?
A. 20 B. 10
C. 25 D. 15
E. None of the above
66. By what percent is the number of people who like to play billiards less than the number of members who
like to play squash?
A. 16.67% B. 20%
C. 25% D. 33.33%
E. None of the above
67. The HCF and LCM of two numbers, a and b, are 6 and 180, respectively. The HCF and LCM of b and c
are 30 and 90, respectively. Also, the HCF and LCM of a and c is 6 and 60. Find the product of a, b, and c.
A. 42300 B. 32400
C. 34200 D. 23400
E. None of the above
68. N is the smallest integer such that N/2 is a perfect square and N/3 is a perfect cube. The sum of factors of
N is:
A. 1518 B. 1185
C. 1815 D. 1851
E. 1581
69. How many palindromes can be formed using three or more letters of the word palindrome?
www.byjusexamprep.com

A. 100000 B. 200000
C. 222200 D. 222000
E. 220000
70. At what time between 8 and 9 o’ clock are the hands of a clock 4 minutes apart?
A. 8 : 51 B. 8 : 48
C. 8 : D. 8 : 41
E. Both B and C
71. Direction: Read the following passage and answer the questions.

*Last 5 Matches (Recent First) means that the results for matches are arranged according to their order of
occurrence from left to right such that the most recent match is denoted in the first column.

Group league matches consisting of 5 rounds of WCC Men’s T20 Cricket World Cup 2022 was held
recently. One of the two groups had teams – B, I, N, P, S and Z. Each team played with every other team
once in each of the 5 rounds. A match either ended in a win (W) or a loss (L) or the match was abandoned
due to rain (NR: No record). Any team that won was awarded 2 points, while a team that lost was awarded
0 points. The two teams of the abandoned matches were awarded 1 point each.

The table given below gives the details of the matches played between the teams and the final points at
the end of the group league matches. Some of the details are missing and are left blank. The additional
information known is as follows:
1. S played its second match against B and its third match against I.

2. P played its second match against Z and its third match against N.

3. Z lost its last match to I.


www.byjusexamprep.com

7. The 4th match of N was played against which team?


A. I B. S
C. B D. Z
E. P
72. The 1st match of B was played against which team?
A. N B. I
C. P D. S
E. Cannot be determined
73. Which of the following is true?
A. The first match of I was against B. B. The fourth match of P was against I.
C. The first match of P was against I. D. The fifth match of S was against Z.
E. The third match of P was against I.
74. The ratio of the number of diagonals in regular polygons (with least number of sides) P1 and P2 is 4:7.
Find the ratio of each interior angle of these two given polygons.
A. 2:3 B. 3:2
C. 4:5 D. 5:4
E. 2:5
75. The sum of the marks obtained by Prasad and Pavan exceeds the maximum marks by 30%. If the passing
marks are 9.09% more than the marks obtained by Prasad and 20% less than the marks obtained by
Pavan, then find the percentage of passing marks.
A. 62.5% B. 56.25%
C. 60% D. 50%
E. 44.44%
www.byjusexamprep.com

Solutions

1. E
Sol.

Passage summary: The passage reviews a book by Tricia Hersey Rest is Resistance. It
discusses the factors that have shaped her perspective that through rest, we can reclaim our
bodies and our capability to imagine alternative life scenarios.

Genre: Sociology/Literature

Word count: 350

Question type: Specific Idea Based Question

Refer, ‘Her perspective is heavily influenced by black feminist traditions, Afrofuturist thought
and, above all, the Pentecostal church in which she was raised (her father was an assistant
pastor). Indeed, Rest is Resistance, a passionate polemic for the most part, reads at times like
a sermon or a prayer.’

Thus, the author lists three influences on Hersey’s perspective. The closest that comes as a
reason for the book resembling a sermon or a prayer is her upbringing.

So, A and B are eliminated.

C has not been mentioned in the passage in the context of the book resembling a prayer.

D is distorted. She did not actually work as a Bishop. It was just a designation that she created
for herself in her project. (Notice the double quotes around Bishop.)

2. D
Sol.
www.byjusexamprep.com

Passage summary: The passage reviews a book by Tricia Hersey Rest is Resistance. It
discusses the factors that have shaped her perspective that through rest, we can reclaim our
bodies and our capability to imagine alternative life scenarios.

Genre: Sociology/Literature

Word count: 350

Question type: Inference

Refer to the last paragraph, particularly ‘Naps and practices of rest reclaim our bodies and
also our stolen “dreamspace”, our capacity to imagine that things could be otherwise.’

A is distorted. The paragraph does not establish any such cause-effect relationship.

B is broad. We are specifically talking about our body and the ability to imagine alternative
scenarios.

C distorts the sentence and does not give its implication.

D is the closest to the paragraph’s intent.

E talks about the soul, which is alien to the passage.

3. D
Sol.

Passage summary: The passage reviews a book by Tricia Hersey Rest is Resistance. It
discusses the factors that have shaped her perspective that through rest, we can reclaim our
bodies and our capability to imagine alternative life scenarios.

Genre: Sociology/Literature
www.byjusexamprep.com

Word count: 350

Question type: Inference

A can be inferred from ‘But there are limitations to rest as a form of subversion, too; all that
Winston manages in the end is to carve out tiny moments of freedom before he is inevitably
crushed.’ The words ‘all that’ and ‘tiny’ show that rest did not help Winston much.

B can also be inferred from ‘Just as the slave’s body is not their own, so too does capitalism
extend ownership over all our bodies, albeit in more insidious and disguised ways.’ This
highlights the similarity between white supremacy and capitalism.

Refer, ‘the exhausted self—shorn of the energy to do anything other than comply—is a
malleable self.’ 3 can be inferred from this.

D is inaccurate. The book uses the ideas of power exploiting our bodies and energy. This does
not imply that the entire book is analogous to Nineteen Eighty Four.

E can be inferred from the first paragraph, from ‘…unprogrammed time may be liberating’ and
‘all that Winston manages in the end is to carve out tiny moments of freedom..’. Also, refer to
the last sentence of the passage.

4. C
Sol.

Let’s take a moment to appreciate the twist in this poem. In the first two lines, the relationship discussed
comes across as loving. The hook-eye enclosure is the one used to fasten garments together. So, the two
people just fit.

But the next two lines reveal a twist. The hook is not the garment hook, but a fish hook stuck in an open
eye. It invokes disturbing and violent imagery. This indicates that their relationship is in reality a painful
one. Hence, C is the correct answer

A can be easily eliminated.


www.byjusexamprep.com

B is close, but the idea is not that the relationship is occasionally a happy one. Rather, it ‘appears’ happy
on the outside, but isn’t.

D cannot be inferred as the violence could be emotional or mental.

E is not substantiated by the poem. It seems like one person is suffering more here, the one who has an
open eye.
5. D
Sol. ‘You’ fit into ‘me’; ‘A’ fish hook; ‘An’ open eye—these are clues that this poem talks about two people. So,
1 follows.

3 can be inferred as the eye in the second line is most likely the eye of the hook-eye enclosure, while the
eye in the fourth line is the eye of a fish. Hence, 3 follows.

2 does not follow as a fish hook and hook-eye hook are not the same.

Hence, D is the correct answer.


6. A
Sol.

A is the correct answer because there are errors in 2, 3, and 4.


In sentence 2, ‘had’ should be replaced by ‘has’ because the second part of the sentence is in the present
tense.
In sentence 3, there should be an article ‘a’ before testimony and testimony should be followed by ‘to’.
‘For’ has been used incorrectly in this sentence.
In sentence 4, there is a parallelism error. ‘Call’, ‘apologise’, ‘accept’ and ‘offer’ should be in the first form
of the verb.

7. E
Sol. Let’s read the entire paragraph to understand the context. Let’s look at the second blank first.
The second blank is followed by ‘to’. We can eliminate options A and B as ‘idea to’ doesn’t
make sense. We can also eliminate option C as ‘imperative to’ is incorrect. Option D can also
be eliminated. Hence, the correct answer is option E.
www.byjusexamprep.com

8. C
Sol.
This question can be easily solved with the help of mandatory pairs.
5 and 3 form a pair as 5 talks about a limitation and 3 explains that limitation.
3 and 2 form a mandatory pair as 3 ends with the idea ‘scepticism can never be refuted’ and 2
talks about the beginning of all refutation.
1 follows 2. Note the use of ‘this destructive kind’.
4 comes at the end as it continues with the idea in 1.
Hence, the correct answer is option C (53214).
The correct paragraph reads as follows:
When we speak of philosophy as a criticism of knowledge, it is necessary to impose a certain
limitation. If we adopt the attitude of the complete sceptic, placing ourselves wholly outside all
knowledge, and asking, from this outside position, to be compelled to return within the circle of
knowledge, we are demanding what is impossible, and our scepticism can never be refuted.
For all refutation must begin with some piece of knowledge which the disputants share; from
blank doubt, no argument can begin. Hence, the criticism of knowledge which philosophy
employs must not be of this destructive kind if any result is to be achieved. Against this
absolute scepticism, no logical argument can be advanced.

9. A
Sol.
We need to look at clues here before figuring out the first or the last sentence.
4 and 5 form a mandatory pair as 4 talks about ‘possibility of sensible things being both like in one way
and unlike in another’ and 5 provides an example for this phenomenon. 3 follows 5 as it carries forward
the same idea. 2 comes at the end as it concludes the discussion with ‘Hence’. 1 should come at the
beginning as no other sentence talks about ‘Simmias’ and 4 follows 1 as it draws an inference from 1.
Hence, the correct answer is A (14532).
The correct paragraph reads as follows:
Socrates points out that Simmias is taller than Socrates (and hence tall), but that Simmias is also shorter
than Phaedo (and hence short). Clearly Socrates envisages the possibility of sensible things being both
www.byjusexamprep.com

like in one way and unlike in another. For example, Socrates is like Plato (in that each is a philosopher)
and unlike Meletus (in that one, but not the other, is a poet). Being like Plato, Socrates is like; being unlike
Meletus, Socrates is unlike. Hence, Socrates is both like and unlike.

10. C
Sol.

Passage summary: The passage discusses increased loneliness in today’s world because of a decrease
in friendships and associates it with an impulse for totalitarianism.

Genre: Culture

Word count: 383

Question type: Specific Idea

A is false. The Christians did not get revolution, and the workers did not get God.

B and D are not substantiated by the passage. The church and party may or may not be proxies or
extensions for God and revolution.

C can be inferred. Notice the use of ‘Instead’. This means that the groups did not get what they wanted

E is extreme. The things that they got may not be opposite, but just different from what they wanted.
11. C
Sol. Passage summary: The passage discusses increased loneliness in today’s world because of a decrease
in friendships and associates it with an impulse for totalitarianism.

Genre: Culture

Word count: 383

Question type: Specific Idea


www.byjusexamprep.com

A is alien to the passage. The passage does not talk about any dispute with friends.

The passage does not define the term friendship recession only in relation to one factor. So, B and D are
eliminated.

E describes the effect of friendship recession, not the term itself.

C is an apt description and is substantiated by the first two sentences of the passage.
12. D
Sol. Passage summary: The passage discusses increased loneliness in today’s world because of a decrease
in friendships and associates it with an impulse for totalitarianism.

Genre: Culture

Word count: 383

Question type: Critical reasoning

The argument made by the author towards the end of the last paragraph is:

‘As Americans become lonelier and more isolated in the new century, the same totalitarian temptation now
lurks.’

It is an analogical argument that says that what happened in the twentieth century is likely to repeat itself.
In saying that, it assumes that the conditions of the two periods are similar.

So, D weakens it the most.

A does not affect the argument. The author nowhere asserts that loneliness is unique to the twentieth
century.

B does not affect the argument. There could be a number of factors besides loneliness that could lead to a
totalitarian domination, but as long as loneliness is a factor, the argument holds.

C also does not weaken the argument. The current argument is related to Arendt’s. Arendt said that
loneliness leads to a totalitarian impulse when it becomes mainstream. So, she has acknowledged that
loneliness may not always translate to a totalitarian temptation.
www.byjusexamprep.com

E does not weaken the argument because the conditions of America, especially the nature and extent of
its loneliness, could be very different from those of these other countries.
13. D
Sol. We have to identify the option that can be inferred from the information given in the passage.

Option A cannot be inferred as the passage mentions the protests in Tiananmen square and
the author also does not say that they are not likely to succeed.

Option B cannot be inferred as the response or the attitude of the government has not been
mentioned in the passage.

Option C cannot be inferred as we only know that there were protests in Tiananmen square.
We cannot infer whether this protest is a repetition of what happened earlier.

Option D is correct. Refer to the last few lines of the passage:

… the protesters in China have made their voices heard under very difficult conditions. They
demonstrated the value of protest and dissent — freedoms that many in the world hold dear.

Hence, option D is the correct answer.

14. C
Sol.
Read the entire paragraph before looking for words to fill in the blanks. These questions test
both grammar and vocabulary. We need an adjective in the first blank. Therefore, we can
eliminate ‘pharaoh’ as it is a noun. Now, we are left with options C, D, and E. We need an
adjective which reflects positively about civilisation. ‘Infinitesimal’ means ‘very small in size’
and can therefore be rejected. ‘Latest’ civilisation is logically incorrect. Pharaonic is the correct
word for the first blank. This means C has to be the correct answer.
For the second blank, we need a word which signifies ‘in- depth’. ‘Profound’ is the correct
www.byjusexamprep.com

word.
For the third blank, which means ‘to display’. ‘Exuded’ is the correct word.
Third blank requires a word which means heirs. ‘Inheritors’ is the correct word.

15. A
Sol. The author’s main point is stated in the last few lines of the paragraph:

Casting human agency in a more distributive and collective mode will help us to become more
genuinely autonomous—as interwoven, interdependent, and relational beings.

Hence, we can infer option A.

Option B is incorrect as it distorts the meaning of the first line of the paragraph:

Thinking of human agency as an individual and independent activity is misguided.

Option C is incorrect as it is vague. The author says that ‘ … by acknowledging that we aren’t
in full control of our actions, that we gain more control over their outcomes …’

Option D misrepresents the meaning of the following sentence in the paragraph. Hence, it
cannot be inferred. Refer to these lines:

In order to more justly attribute responsibility, guilt and merit across the field of entangled
actors, and to design social systems that offer more equitable and nurturing paths for the
future, we must look to the others around us who participate in our agency.

E is a generalisation of the facts given in the passage and cannot be called a proper
conclusion.

Hence, option A is the correct answer.


www.byjusexamprep.com

16. C
Sol. The key trigger here is that the ‘he’ mentioned in the context gets tired of arguing with another person who
agreed with everything he said. This means that he is the one who likes or enjoys getting into arguments.
We can eliminate options A, B, and D as the first words are positive adjectives and do not come close to
the requirement here. Now, option E has a problem. It would not take a long time for a quarrelsome
person to become tired with somebody who always agrees. He also would not feel victorious after such an
incident. All these point to option C as the answer.
17. D
Sol.

Passage summary: The passage draws parallels between the tradition of coffeehouses and
the social media networks today.

Genre: Culture/Technology

Word count: 400

Question type: Specific idea

Refer to the sentences at the very beginning of this passage. The purpose of the author is to
highlight that even though it may seem unlikely, there are similarities between the coffeehouse
tradition and the social media networks of today. Hence, D is the correct answer.

A is narrow and is not the purpose of the author. Also, the history of only coffeehouses has
been discussed in the passage.

B is incorrect in saying that the author is trying to clarify a concept.

C is again narrow. The debates over two spaces have been discussed to show their similarity.

E is extreme and goes beyond the scope of the passage in saying that an online space has
‘replaced’ a traditional space.
www.byjusexamprep.com

18. D
Sol.

Passage summary: The passage draws parallels between the tradition of coffeehouses and
the social media networks today.

Genre: Culture/Technology

Word count: 400

Question type: Inference

I cannot be inferred. Refer to the sentence, ‘Critics deemed them time-wasting, seditious,
boastful, unmanly, mob-minded and intellectually unserious.’ So, it was only the opinion of
critics. We can’t say that this is true on the basis of the passage.

II is distorted in presenting social media as negative in this respect when the passage
presents it in a positive light. Refer ‘Platforms such as Twitter and Facebook have helped
campaigners and revolutionaries spread their messages to others—from the Arab Spring to
the Black Lives Matter movement, and the ongoing Iranian protests today.’

III is too broad. As per the passage, the two spaces are similar with respect to news, views,
and debates. We cannot generalise this. Also, the passage discusses coffeehouses of the
past.

19. E
Sol.

Passage summary: The passage draws parallels between the tradition of coffeehouses and
the social media networks today.

Genre: Culture/Technology
www.byjusexamprep.com

Word count: 400

Question type: Structure based question

Refer to the sentences, ‘Coffeehouses specialised in certain fields of interest or trade: lawyers,
printers, merchants, and insurers all had their favoured locales in 18th-century London. Some
even developed into private exchanges and clubs; the London Stock Exchange and Lloyd’s of
London both began as coffeehouses. But most were fundamentally open and democratic,
albeit male-centric, allowing in any man who could pay the price of a coffee.’

A is false as per the sentence, ‘But most…coffee.’

B is mentioned later to highlight that these exchanges and clubs were exceptions to the norm.

C is generalising what happened with some coffeehouses.

D is distorted. The examples come later. They are not the purpose behind mentioning the
terms.

E is the best fit. Private exchanges and clubs serve as examples of what some coffeehouses
evolved into as they were specialised in certain fields of trade and interest.

20. A
Sol.

Passage summary: The passage argues that wisdom is not separate from the question of cultural identity,
rather it is about figuring out how humans can live well and develop universal standards of well-being,
while also knowing that no single standard of wellness will ever be meaningful to everyone. To build this
argument, it talks about moral groundedness and metacognition.

Genre: Philosophy

Word count: 361

Question type: Main Idea


www.byjusexamprep.com

Refer to the sentence, ‘Wisdom in this sense is not separate from identity, but is about figuring out how
humans can live well and develop universal standards of well-being, while also knowing that no single
standard of wellness will ever be meaningful to everyone.’ It sums up the point the author is trying to make
in the passage.

The author’s message is about balancing our shared humanity ( through moral groundedness) and our
differences (through metacognition). Thus, A is the most apt.

B and E focus only on the shared humanity aspect.

C is unrelated.

D focuses only on the aspect of metacognition.


21. B
Sol. Passage summary: The passage argues that wisdom is not separate from the question of cultural identity,
rather it is about figuring out how humans can live well and develop universal standards of well-being,
while also knowing that no single standard of wellness will ever be meaningful to everyone. To build this
argument, it talks about moral groundedness and metacognition.

Genre: Philosophy

Word count: 361

Question type: Direct

A is false as per the passage. Refer, ‘Perhaps surprisingly, this challenge to unite shared humanity and
cultural identity is not unique to the modern world.’

B is true as the passage talks about a current definition of wisdom.

C is distorted in saying that they are indispensable in attaining wisdom. They are just 2 key features as per
a definition. Refer, ‘The two key features of the current definition are: moral groundedness and
metacognition.’
www.byjusexamprep.com

D is the argument that the author talks about in the beginning. The author then builds a counterargument
to this.

E is flawed in its alien comparison with the laws of nature.


22. B
Sol. Passage summary: The passage argues that wisdom is not separate from the question of cultural identity,
rather it is about figuring out how humans can live well and develop universal standards of well-being,
while also knowing that no single standard of wellness will ever be meaningful to everyone. To build this
argument, it talks about moral groundedness and metacognition.

Genre: Philosophy

Word count: 361

Question type: Further application

A is in line with moral groundedness, as discussed in the passage.

B is against the author’s opinion: ‘Wisdom in this sense is not separate from identity, but is about figuring
out how humans can live well and develop universal standards of well-being, while also knowing that no
single standard of wellness will ever be meaningful to everyone.”

C also aligns with the author’s views.The author is likely to support C as it furthers the understanding of
wisdom.

D will also be supported as this quest is mentioned in Para 1.

E is in line with the passage, especially its last sentence.


23. C
Sol. Option A cannot be concluded or inferred as the reasons for air travel escaping carbon pricing
have not been mentioned. Moreover, the airlines are responsible for a quarter of all emissions,
so we cannot call the emissions ‘negligible’.

From the given information, we cannot infer anything about the EU dragging its feet. We also
cannot know for sure that the present scheme is good but not likely to meet the 2050 target.
www.byjusexamprep.com

The EU may have come up with the scheme only now. So, we cannot infer that it has delayed
implementing the scheme. Therefore, option B is eliminated.

Option D is the author’s advice. We cannot conclude anything about what the EU should do to
meet its target. The passage only mentions that including airlines in the carbon pricing scheme
is unlikely to help it meet its zero aviation emissions target by 2050.

Option E is out of scope of the passage as the reasons for airlines escaping carbon pricing so
far have not been mentioned or even implied.

Option C is correct. The author says that including airlines in the carbon pricing scheme is a
step in the right direction but is unlikely to achieve net zero aviation emissions by 2050. Based
on this, we can infer that with this step sometime after 2050, the target may be met.

Hence, option C is the correct answer.

24. D
Sol.
Option A is implicit in the first few lines of the passage.

‘Camouflage is the by-product of an evolutionary arms race between one group—the


predators—that want to eat another group—the prey.’

Option B is implicit in the following lines:

‘The predators look to eat by breaking through the camouflage of the prey.’

Option C is implicit in the following lines:

‘Camouflage is the by-product of an evolutionary arms race between one group—the


predators—that want to eat another group—the prey.’
www.byjusexamprep.com

Option E is implicit in the fact that the predator tries to break through the camouflage and the
prey tries to survive by tricking the perceptual and cognitive systems of the predators.

Option D cannot be called an assumption. It contradicts the statement that manifestations


vary. Also ‘limitations’ has not been mentioned.

Hence, option D is the correct answer.

25. E
Sol. Option A cannot be concluded as nothing in the passage suggests that managers don’t have
the trait while the rest of the population does.

Option B has an extreme word, i.e., ‘crucial’.

Option C is close but incorrect. Managers are trained to use deliberative thinking and more
and more processes are designed to stamp out instinctive thinking does not mean managers
are trained to kill their instinct.

Option D is not correct as there is no comparison between which is better.

Option E is the correct answer. The second part of the paragraph says that instinct is
important in certain situations and the manager has to take a call based on their instinct.

Hence, option E is the correct answer.

26. C
Sol.
Option C is the correct answer because there are errors in 3, 4, and 6.
3: The correct phrase is ‘points of view’ and not ‘point of views’.
4: ‘Yet’ is not the correct word to be used here. ‘If’ is the correct word as there is a condition
here.
6: Work is not countable. So, it should be followed by ‘is’.
www.byjusexamprep.com

27. B
Sol. Let’s look at all the options.

A: This option might dissuade Raveena from setting up the stall because paying Rs. 50,000 every month
for a new business might be difficult.

B: This option might persuade Raveena to set up a stall in the college as no other stalls in the college
serve chicken sandwiches. She might want to tap this market.

C: This option might dissuade Raveena from setting up the stall because making an investment Rs. 5
lakhs for a new venture might not be a very good idea.

D: This option might dissuade Raveena from setting up the stall because the cheapest dish she sells at
her van is priced above Rs. 55.

E: This option might dissuade Raveena from setting up the stall because it might be difficult to persuade
the students to switch from their favourite stalls.
28. D
Sol. First let’s figure out which of the given steps can be taken.

I. A police complaint does not make sense because nothing has been proven yet. If Akshay is proven
guilty, it might lead to some unwanted things for Raveena.

II. Asking someone else to keep an eye is not advisable as this will give a bad name to Akshay.

III. Discussing the issue with Akshay is the most logical thing to do as nothing has been proven yet. The
discussion might lead to some fruitful results.

IV. Installing CCTV cameras will help Raveena take stock of the situation.

V. Raveena might catch Akshay red-handed if she visits the stall randomly.
29. E
Sol. None of the given courses of action are logical because of the following:

I. Asking someone to leave now because they won’t be available next month is foolish as it means loss of
almost a month’s business.
www.byjusexamprep.com

II. Offering a bonus to Akshay to work during his exams is insensitive.

III. Postponing the exams for one person does not make sense.
30. B
Sol.

In order to help the man admit his wife to the hospital, writing an email or tweeting won’t help as it would
take a lot of time. Hence, 2 is out of question. 3 cannot be considered because it involves threatening.

5, 4, and 1 can actually yield results. The first step is to approach the people at the reception, then the
doctors in the emergency department and finally, the CMO. All these people are present in the hospital at
any given point of time.
31. D
Sol. Setting up beds for the needy on the footpath outside the hospital does not help as it is still discrimination
and won’t help improve anyone’s health.

Setting up health camps on a first-come-first-serve basis in the hospital premises won’t help because
anyone can use these services.

Providing a 10% discount won’t help anyone much.

Providing free fruits and fruit juices to children is just a simple act of charity.

Hence, the correct answer is option D.


32. A
Sol. You should stick to the original comments as that is the truth you saw. You do not have to change or
delete anything because someone else wants you to. Hence, the correct answer is option A.
33. D
Sol. The reason for coming late and absenteeism of the old-timers is the single-handed decision–making by
Mr. Suraj as he does not take anyone’s advice or involves anyone in the decision-making process. Hence,
the answer is option D.

Had option A been the problem, Mr. Suraj would not have been appointed as the Managing Director.
www.byjusexamprep.com

Six fresh MBA graduates being appointed as branch managers is a decision taken by Mr. Suraj. So, this
cannot be the answer.

Options C and E have not been mentioned or hinted at.


34. E
Sol. Going on leave and resigning are both not advisable as one should not evade their responsibilities.
Therefore, A and D are out of scope.

B and C are wrong because offering perks and salary hikes to get things done is unethical.

Hence, the correct answer is E.


35. A
Sol. The best combination of actions would be to take suggestions from employees, seek ideas from BoD
members, and take a collective decision.

1 is too extreme and won’t solve the problem. 5 is not the right course of action as that would mean
running away from one’s responsibilities.
36. D
Sol.
Since you are just studying the past years’ data, it is advisable to let your manager know about
any discrepancies and seek her approval for anything you want to do. You cannot go around
talking about any issues with anyone you want to or start reaching out to heads of
departments for no reason. Hence, the correct answer is option D.
37. C
Sol.
The first thing the director should have done is report the matter to the sexual harassment
prevention committee or the HR department. Doing anything or taking decisions on one’s own
is not the right way of dealing with such issues. Hence, option C is the correct answer.
38. D
Sol.

This is a tricky question where it’s important to know that one should let the HR know if they indulge in a
romantic relationship at the workplace, more so within the same team.
www.byjusexamprep.com

In this case, the first thing you would do is discuss the issue with the sexual harassment prevention team.
Then issue a show cause notice to Ms. Nupur who did not inform the HR regarding her romantic
relationship with Mr. Anshul and tried to take advantage of it. A show cause notice will be issued to Mr.
Anshul and Ms. Wolowitz as they are also into a romantic relationship.

39. D
Sol. Let’s consider each of the courses of action.

I. Ask Anju to send an apology mail to him marking the whole team if he wants him to conduct the session:
This is unfair to Anju as sending an apology mail marking the whole team is humiliating.

II. Get the session timings adjusted so that the timings do not clash with the tournament and take the
session: This is the right thing to do as this might solve the problem.

III. Call Sachin and tell him that Anju is fond of causing inconvenience to him: This is not the right course
of action as generalising Anju’s behaviour based on one instance is unfair.

IV. Have a meeting with Sachin and Anju to clarify everything: This is logical as it will clarify things for
everyone.
40. A
Sol. A is the only option which will be fruitful. The promised session will be conducted on time which is the
business’s and the student’s requirement.

B: Calling Anju is unfair as she is on a sabbatical.

C: Not doing anything is not an option as it will affect the business.

D: Threatening is unfair.

E: Connecting with students via their personal accounts might not be permitted by the company.
41. C
Sol. Only II and III can help Anju resolve the issue. Let’s take a look.

I. Wait for the student to escalate the issue further and Tamanna to work: This will just delay things and the
issue might not get resolved at all.
www.byjusexamprep.com

II. Write a formal email to Tamanna, marking Sachin, and persuade her to get the issue resolved: A formal
email might do the trick. Also, looping in Sachin is of utmost importance.

III. Reach out to Sachin to understand Tamanna’s roles and responsibilities: It’s a good idea to understand
a person’s roles and responsibilities before pointing fingers.

IV. Share Tamanna’s email ID with the student and ask them to write a stern email to Tamanna: This will
bring bad name to the company.
42. D
Sol. The situation did not become unmanageable with Ramesh’s unfortunate demise since the doctor
explained his stand adequately. Thus, statement III has to be eliminated.

Statement II can also be eliminated as Ramesh was adequately tested at RMNI hospital.

It cannot be determined from the passage whether statement V is true.

The situation became unmanageable when the hospital fired the doctor based on social media reactions
and posted this bit of news on social media. Thus, statements I and IV are correct.

Hence, the correct answer is option D.


43. C
Sol. Note that the passage states that ‘certain types of heart attacks are virtually undetectable’. The hospital
should try and ascertain as to the type of heart attack Ramesh suffered before taking a decision on
reinstating the doctor to his former position. Thus, option C is the best way forward.

Prior to ascertaining the type of heart attack suffered by Ramesh, the hospital should neither apologise
nor reinstate the doctor. Eliminate options A and B.

Option D would not be fair to the doctor and option E makes no sense as the public reaction on social
media is outside the control of the hospital.

Hence, the correct answer is option C.


44. B
Sol. The best course of action would have been to call cardiologists as it is not a speciality heart hospital.
Expert guidance could have saved Ramesh.
www.byjusexamprep.com

Option A is not correct. Since Ramesh has been brought to his hospital, the doctor had no choice but to
examine him.

Option C is incorrect because it’s difficult to say if the seniors would have been of any help as it’s not a
speciality heart hospital.

Option D is incorrect because there was no point referring to another hospital as the results indicated that
there was no danger.

Option E would have made the case against the doctor more serious. His hospital would not have spared
him had he made such a statement.

Hence, the correct answer is option B.

45. B
Sol. Statement I is a valid reason as drought-like conditions would inhibit the growth of oilseed crops.
Statement II is not valid for strengthening the case for the Oilseeds Board since that points to faulty
communication of top officials of the Board. Statement III strengthens the case for the board officials since
loans are required by farmers to buy seeds, equipment, and more. Statements IV and V are not valid
cases for strengthening the board officials’ stand of not meeting targets since there are a few assumptions
regarding the way of cropping and farming involved in these two statements. Therefore, the valid
statements are I and III.

Hence, the correct answer is option B.


46. D
Sol. Statements A, B, and C are generic motivational tactics that would not impact the officials of the Oilseeds
Board. They can be eliminated. Statement E would probably demoralise the officials further instead of
motivating them. Only D would probably motivate the officials since it impacts directly on their financial
well-being.

Hence, the correct answer is option D.


47. B
Sol. The passage states that only an average of 30 million tons was achieved for the first four years of the
plan. It may be that some years could have been less than 28 million tons and some years more (to arrive
www.byjusexamprep.com

at the average of 30 million). Secondly, without knowing weather conditions for the fifth year and other
factors, such as availability of loans to farmers, we cannot say for sure what the total production will be.
Drought-like conditions may bring the total down to below 28 million tons while excessive rains may also
destroy crops. The final figure cannot be determined from the data in the passage. Thus, it cannot be said
that the target of 28 million tons for the fifth year will be definitely met.

Hence, the correct answer is option B.


48. A
Sol. The remainder when is divided by x – 1 is

, that is 1

… (1)

The remainder when when divided by x – 2 is

is 7

……(2)

will give the following:

Hence, option A is the correct answer.


49. C
Sol.

Squaring the values on both sides,


www.byjusexamprep.com

Cubing on both the sides, we get:

+ +3 ( )

+ + 3( )

+ +

Hence, option (C) is the correct answer.


50. B
Sol. Let C be the point at which PB touches the circle and O is the centre of the circle.
www.byjusexamprep.com

Given:

PC = CB and OA = OB = 4 units
As length of median to the hypotenuse is equal to half the length of the hypotenuse

PC = CB = CA and As ∠AOB = 180o then ∠ACB = 90o


Using the Pythagorean theorem, or or

Again using the Pythagorean theorem, or

or units.
Area of triangle PAB sq units.

Hence, option B is the correct answer.


51. B
Sol. Given, log3(24 + log5x) = 3

24 + log5x = 33 = 27
log5x = 27 – 24 = 3

x = 53 = 125
Also, 216log2 64 = 10x + 2log3y + 62
Substituting log2 64 = 6 and x = 125 in the above equation, we get,
216 6 = 10 125 + 2 log3y + 36
1296 = 1250 + 2 log3y + 36
10 = 2 log3y
5 = log3y

y = 35 = 243
y – x = 243 – 125 = 118
Hence, option (B) is the correct answer.
www.byjusexamprep.com

52. D
Sol. (3 – x)(5 – x)(7 – x)…………………..(91 – x)(93 – x) 0

(x – 3)(x – 5)(x – 7)…………………..(x – 91)(x – 93) 0

When x = 3, 5, 7, 9, ……………..91, 93 will satisfy the equality as the value will always be zero.

In (x – 3)(x – 5)(x – 7)………………..(x – 91)(x – 93), there are 46 terms. So, for the expression to be
positive, an even number of terms should be less than 0.

For x = 4, all the terms from (x – 5) to (x – 93) will be negative. Thus, the whole expression will be
negative because there will be 45 negative terms.

For x = 6, all the terms from (x – 7) to (x – 93) will be negative. Thus, the whole expression will be positive
because there will be 44 negative terms.

For x = 8, all the terms from (x – 9) to (x – 93) will be negative. Thus, the whole expression will be
negative because there will be 43 negative terms.

For x = 10, all the terms from (x – 11) to (x – 93) will be negative. Thus, the whole expression will be
positive because there will be 42 negative terms.

Similarly, at x = 14, 18, 22, 26, 30, 34, 38, 42, 46, 50, 54, 58, 62, 66, 70, 74, 78, 82, 86, 90 the given
expression will be positive.

There are 22 such values of x for which the expression is positive.

There are 46 values of x for which the expression becomes 0.

Thus, there are 68 solutions.

Hence, option D is the correct answer.


53. D
Sol. Total numbers possible = All two-digit numbers till 150 + All three-digit numbers till 150
= 90 + 51 = 141
Two-digit numbers divisible by 9 = 18, 27, 36, 45, 54, 63, 72, 81, 90, 99, i.e., total 10 numbers
www.byjusexamprep.com

So, favourable cases = 90 – 10 = 80

Hence, the required probability =


Hence, option D is the correct answer.
54. C
Sol. Let T be the time required to finish the work when all the three are working.

Amar and Akbar take 4T and T + 2 hours, respectively, while working individually to complete the entire
task.
Antony will take T + 6 to complete the task individually.
Then,

Thus,

or

The ratio of the time = 8 : 4 : 8 = 2 : 1 : 2

Ratio of their efficiencies

The money received by Akbar

Hence, option C is the correct answer.


55. C
Sol.

As is always divisible by a + b when n is odd

must be divisible by

Hence, option C is the correct answer.


www.byjusexamprep.com

56. D
Sol. Let the distance covered by the person on the bicycle be x km.

So, the distance covered by the person by bike is (144 – x) km.

According to the question, we can write the equation as follows:

13x + 720 – 5x = 1040

⇒ 8x = 320

⇒ x = 40

So, the person covered a distance of 40 km on bicycle.

Hence, option (D) is the correct answer.


57. A
Sol. 20% of 430 = 86

86 = 53 + 16 + 9 + 4 + 3 + 1

So, Option A is correct.

58. A
Sol. 50% of 430 = 215

215 = 76 + 59 + 53 + 27

So, Option A is correct.

59. B
Sol. 20% of 430 = 86

It can happen in the following ways:

83 + 3
www.byjusexamprep.com

76 + 7 + 3

76 + 9 + 1

59 + 27

59 + 16 + 7 + 4

59 + 16 + 7 + 3 + 1

53 + 16 + 9 + 7 + 1

53 + 16 + 9 + 4 + 3 + 1

So, option B is correct.

60. C
Sol. As per the given information:

The value of n has to be greater than or equal to 7.

Case 1:

When n = 7,

Total number of ways which is far away from the given value

Case 2:

When n = 8,

Total number of ways which is far away from the given value

Case 3:
www.byjusexamprep.com

When n = 9, there will be two items in the fourth group.

Total number of ways which is far away from the given value

Case 4:

When n = 10, there will be three items in the fourth group.

Total number of ways which is far away from the given value.

Case 5:

When n = 11, there will be four items in the fourth group.

Total number of ways which is equal to the given value.

So, the value of n is 11 and the value of x is 4.

The value of n + x is 11 + 4 = 15

Hence, option C is the correct answer.


61. D
Sol. There are exactly presentations

There must be one presentation of 12 minutes and one presentation of 6 minutes as minimum and
maximum is given. Given, 4 presentations took 10 minutes or more than 10 minutes. One of them is
already considered above.
Let the three remaining presentations take 10 minutes each.

That is, minutes and 5 presentations are closed.


In the remaining 132 minutes, there can be at most 12 presentations of 10 minutes each and one
presentation of 12 minutes.

Thus, a total of 12 + 3 = 15 presentations of 10 minutes each are possible.

Hence, option D is the correct answer.


www.byjusexamprep.com

62. D
Sol. PS and QT are two of the medians so point G is the centroid of the triangle PQR.
Therefore, PG : GS = 2 : 1 and QG : GT = 2 : 1
From statement I: PG : GT = 2 : 1
GS = GT
QG = PG
From this statement, we cannot find the required area.
From statement II: PQ = 20 cm and QR = 40 cm
We do not know the third side of triangle PQR, Hence, the area of triangle PQR cannot be
found. Hence, the required area cannot be found.

On combining both the statements together,


As GS = GT and QG = PG PS = QT
As both the medians are equal, so the sides on which these medians are drawn must be
equal.
PR = QR = 40 cm
Now, we know all the three sides of the triangle. So, we can find the area of the triangle PQR.
Also, we know that a median divides the area of the triangle into two equal parts. So, the area
of triangle QTR will be half of the area of triangle PQR. Hence, it can be determined.
Therefore, option D is the correct answer.
63. C
Sol. To get the maximum value of n, it is sufficient to find the number of 9 in 181!. Here 9 is not a prime
number. But . Since 3 is a prime number, we will find an IGP of 3 to get the IGP of 9.

The IGP of 3 can be found out as follows:

[181/3] = 60, [60/3] = 20, [20/3] = 6, [6/3] = 2

IGP of 3 is 60 + 20 + 6 + 2 = 88

Hence, the IGP of 9 is


www.byjusexamprep.com

Therefore, the maximum value of n so that is always divisible 181! is 44.

Hence, option C is the correct answer.


64. B
Sol. Let the number of members who like to play only squash be 4x; so, the number of members
who like to play only snooker will be x.

Also, let the number of members who like to play all the three games be y, which is also equal
to the number of members who do not like any of the three games.

The number of members who like to play only billiards and snooker and the number of
members who like to play only snooker and each of them is equal to twice the number of
members who do not like to play any of the three games, i.e., x = 2y.
www.byjusexamprep.com

The number of people who like to play only billiards and squash is the same as the number of

members who like to play only squash and snooker( ), which is equal to half the number of
members who like to play billiards and snooker, i.e., 3y.

One-third of the members who like to play billiards also like to play snooker.

Let the number of members who like to play billiards be z, so z/3 = 3y or z = 9y.
www.byjusexamprep.com

From the final venn diagram, the number of members who play squash = 12y = 48, so y = 4

Thus, the number of members who like to play only billiards is = 18.

Hence, option B is the correct answer.

65. A
Sol. Let the number of members who like to play only squash be 4x; so, the number of members
who like to play only snooker will be x.

Also, let the number of members who like to play all the three games be y, which is also equal
to the number of members who do not like any of the three games.
www.byjusexamprep.com

The number of members who like to play only billiards and snooker and the number of
members who like to play only snooker and each of them is equal to twice the number of
members who do not like to play any of the three games, i.e., x = 2y.

The number of people who like to play only billiards and squash is the same as the number of

members who like to play only squash and snooker( ), which is equal to half the number of
members who like to play billiards and snooker, i.e., 3y.
www.byjusexamprep.com

One-third of the members who like to play billiards also like to play snooker.

Let the number of members who like to play billiards be z, so z/3 = 3y or z = 9y.

From the final venn diagram, the total number of members in the Marvel club is or y
= 20, which is equal to the number of members who like to play all three games.

Hence, option A is the correct answer.


www.byjusexamprep.com

66. C
Sol. Let the number of members who like to play only squash be 4x; so, the number of members
who like to play only snooker will be x.

Also, let the number of members who like to play all the three games be y, which is also equal
to the number of members who do not like any of the three games.

The number of members who like to play only billiards and snooker and the number of
members who like to play only snooker and each of them is equal to twice the number of
members who do not like to play any of the three games, i.e., x = 2y.
www.byjusexamprep.com

The number of people who like to play only billiards and squash is the same as the number of

members who like to play only squash and snooker( ), which is equal to half the number of
members who like to play billiards and snooker, i.e., 3y.

One-third of the members who like to play billiards also like to play snooker.

Let the number of members who like to play billiards be z, so z/3 = 3y or z = 9y.
www.byjusexamprep.com

From the final venn diagram, the desired percentage is = 25%. Hence, option C
is the correct answer.

67. B
Sol. Let a and b are 6x and 6y, respectively.

The possible values of (x, y) are (1, 30), (2, 15), (3, 10), (5,6)

The possible values of (a, b) are (6, 180), (12, 90), (18, 60), (30, 36)

Let b and c are 30p and 30q, respectively.

The possible values of (p, q) is (1, 3)

The possible values of (b, c) is (30, 90)

From the above two cases, the possible values of (a, b, c) are (12, 90, 30) and (36, 30, 90).

The only possibility to get the HCF and LCM of a and c as 6 and 60 is (12, 90, 30)

The required product is

Hence, option B is the correct answer.


68. C
Sol. Let and
www.byjusexamprep.com

For N to be an integer, 6q must be a perfect square number. So the smallest possible value of q is 6.

Sum of factors of N

Hence, option C is the correct answer.


69. C
Sol. The word palindrome contains 10 different alphabets, viz., P, A, L, I, N, D, R, O, M, E.

Three-lettered palindromes can be formed in ways

Four-lettered palindromes can be formed in ways

Five-lettered palindromes can be formed in ways

Six-lettered palindromes can be formed in ways

Seven-lettered palindromes can be formed in ways

Eight-lettered palindromes can be formed in ways

Nine-lettered palindromes can be formed in ways

Ten-lettered palindromes can be formed in ways

Total number of palindromes


www.byjusexamprep.com

Hence, option C is the correct answer.


70. E
Sol. There are 12 divisions in the clock and the angle made at the centre by the 2 divisions =

And each division is 5 minutes apart from the adjacent division.


So, in 5 minutes gap, the angle between the hands of the clock is

Thus, in 4 minutes apart, the angle between the hands of the clock =
Using the formula,

θ = |30H - M|

M= or 48 minutes

So, the time is 8 : or 8 : 48 when the hands of the clock are 4 minutes apart.
Hence, option E is the correct answer.
71. D
Sol. Let’s complete the table first. W, L, and NR stand for win, loss, and no record, respectively.

For every column of the nth matches played, the number of Ws should be the same as the
number of Ls. NR should be in pairs.

The 5th match standings (first from the left) show 3 Ls, so we should have 3 Ws. This means
the remaining cells should be filled with Ws.

The 4th match standings (second from the left) show 3 Ws already. So, the remaining cells
must have Ls.
www.byjusexamprep.com

I has a total of 8 points, and that will be from 4 Ws, which we already have. So, the remaining
cell in row I should be an L.

Now, the 3rd match standings (middle column) should have the remaining cells as Ws as we
already see 3 Ls.

The total points of Z is 3, which so far can come from a W and an NR. The remaining cell of
Z’s row will be a W. So, the 2nd match (fourth column from the left) has 3 Ws already now.
Hence, the remaining cells can be filled with Ls.

S has 5 points and 2 Ws so far, which adds up to 4 points. So, its remaining 1 point will come
from NR, which means the last cell of S’s row will be an NR.

The last column (1st match) has 2 cells left with 2 Ws already present. So, the remaining 2
cells must be Ls.

The points tally from top to bottom will be 8, 6, 5, 4, 4, 3. The final table looks like the one
below.

Let’s now find the sequence of the matches played between the teams. Wn in the cell denotes
the winner’s name (W) in the nth match.

The matrix below will help us figure out the same.


www.byjusexamprep.com

We already know that S played its second match against B and its third match against I, P
played its second match against Z and its third match against N, and Z lost its last match to I.

We can conclude that I won all its matches as it had already lost one match against S (its third
match), and the sole NR match happened between S and Z when they played their first match.

The table so far will look like the one below

The second match was scheduled between P and Z and S and B. So, the second last match
should have been scheduled between I and N.

Similarly, the third match was scheduled between P and N and I and S. The final third match
must have been scheduled between B and Z. So far, Z had already played its 1st, 2nd, 3rd, and
5th matches against S, P, B, and I, respectively (see last row). So, the remaining cell must be
its 4th match where Z must have lost as it had already won one and the other was NR.
www.byjusexamprep.com

The table so far looks as follows:

P was supposed to win 3 matches. It had so far won only one. In the table, two cells are
empty. So, the two empty cells under P should be its wins.

B had to win two matches. So far, B had won only one, and only one match (B vs N) was left.
That should have been a win for B.

N had to win two matches, so far it had won only one. So, its next win should have come from
the last empty cell (N vs S). The table so far will look like the one below.

S had already played its 1st, 2nd, and 3rd matches against Z, B, and I, respectively. Similarly, N
had already played its 2nd, 3rd, and 4th matches against I, P, and Z, respectively. So, the
match between S and N could only be their 5th match.
www.byjusexamprep.com

This concludes that the leftover match of S vs P was S’s 4th match and the leftover match of N
with B was N’s 1st match.

This further concludes that the match between I and P was their first match, the match
between I and B was their 4th match, and the match between B and P was their 5th match.
The final table looks like the one below.

The 4th match of N was played against Z.

72. A
Sol. Let’s complete the table first. W, L, and NR stand for win, loss, and no record, respectively.

For every column of the nth matches played, the number of Ws should be the same as the
number of Ls. NR should be in pairs.

The 5th match standings (first from the left) show 3 Ls, so we should have 3 Ws. This means
the remaining cells should be filled with Ws.

The 4th match standings (second from the left) show 3 Ws already. So, the remaining cells
must have Ls.
www.byjusexamprep.com

I has a total of 8 points, and that will be from 4 Ws, which we already have. So, the remaining
cell in row I should be an L.

Now, the 3rd match standings (middle column) should have the remaining cells as Ws as we
already see 3 Ls.

The total points of Z is 3, which so far can come from a W and an NR. The remaining cell of
Z’s row will be a W. So, the 2nd match (fourth column from the left) has 3 Ws already now.
Hence, the remaining cells can be filled with Ls.

S has 5 points and 2 Ws so far, which adds up to 4 points. So, its remaining 1 point will come
from NR, which means the last cell of S’s row will be an NR.

The last column (1st match) has 2 cells left with 2 Ws already present. So, the remaining 2
cells must be Ls.

The points tally from top to bottom will be 8, 6, 5, 4, 4, 3. The final table looks like the one
below.

Let’s now find the sequence of the matches played between the teams. Wn in the cell denotes
the winner’s name (W) in the nth match.

The matrix below will help us figure out the same.


www.byjusexamprep.com

We already know that S played its second match against B and its third match against I, P
played its second match against Z and its third match against N, and Z lost its last match to I.

We can conclude that I won all its matches as it had already lost one match against S (its third
match), and the sole NR match happened between S and Z when they played their first match.

The table so far will look like the one below

The second match was scheduled between P and Z and S and B. So, the second last match
should have been scheduled between I and N.

Similarly, the third match was scheduled between P and N and I and S. The final third match
must have been scheduled between B and Z. So far, Z had already played its 1st, 2nd, 3rd, and
5th matches against S, P, B, and I, respectively (see last row). So, the remaining cell must be
its 4th match where Z must have lost as it had already won one and the other was NR.
www.byjusexamprep.com

The table so far looks as follows:

P was supposed to win 3 matches. It had so far won only one. In the table, two cells are
empty. So, the two empty cells under P should be its wins.

B had to win two matches. So far, B had won only one, and only one match (B vs N) was left.
That should have been a win for B.

N had to win two matches, so far it had won only one. So, its next win should have come from
the last empty cell (N vs S). The table so far will look like the one below.

S had already played its 1st, 2nd, and 3rd matches against Z, B, and I, respectively. Similarly, N
had already played its 2nd, 3rd, and 4th matches against I, P, and Z, respectively. So, the
match between S and N could only be their 5th match.
www.byjusexamprep.com

This concludes that the leftover match of S vs P was S’s 4th match and the leftover match of N
with B was N’s 1st match.

This further concludes that the match between I and P was their first match, the match
between I and B was their 4th match, and the match between B and P was their 5th match.
The final table looks like the one below.

The 1st match of B was played against N.

73. C
Sol. Let’s complete the table first. W, L, and NR stand for win, loss, and no record, respectively.

For every column of the nth matches played, the number of Ws should be the same as the
number of Ls. NR should be in pairs.

The 5th match standings (first from the left) show 3 Ls, so we should have 3 Ws. This means
the remaining cells should be filled with Ws.

The 4th match standings (second from the left) show 3 Ws already. So, the remaining cells
must have Ls.
www.byjusexamprep.com

I has a total of 8 points, and that will be from 4 Ws, which we already have. So, the remaining
cell in row I should be an L.

Now, the 3rd match standings (middle column) should have the remaining cells as Ws as we
already see 3 Ls.

The total points of Z is 3, which so far can come from a W and an NR. The remaining cell of
Z’s row will be a W. So, the 2nd match (fourth column from the left) has 3 Ws already now.
Hence, the remaining cells can be filled with Ls.

S has 5 points and 2 Ws so far, which adds up to 4 points. So, its remaining 1 point will come
from NR, which means the last cell of S’s row will be an NR.

The last column (1st match) has 2 cells left with 2 Ws already present. So, the remaining 2
cells must be Ls.

The points tally from top to bottom will be 8, 6, 5, 4, 4, 3. The final table looks like the one
below.

Let’s now find the sequence of the matches played between the teams. Wn in the cell denotes
the winner’s name (W) in the nth match.

The matrix below will help us figure out the same.


www.byjusexamprep.com

We already know that S played its second match against B and its third match against I, P
played its second match against Z and its third match against N, and Z lost its last match to I.

We can conclude that I won all its matches as it had already lost one match against S (its third
match), and the sole NR match happened between S and Z when they played their first match.

The table so far will look like the one below

The second match was scheduled between P and Z and S and B. So, the second last match
should have been scheduled between I and N.

Similarly, the third match was scheduled between P and N and I and S. The final third match
must have been scheduled between B and Z. So far, Z had already played its 1st, 2nd, 3rd, and
5th matches against S, P, B, and I, respectively (see last row). So, the remaining cell must be
its 4th match where Z must have lost as it had already won one and the other was NR.
www.byjusexamprep.com

The table so far looks as follows:

P was supposed to win 3 matches. It had so far won only one. In the table, two cells are
empty. So, the two empty cells under P should be its wins.

B had to win two matches. So far, B had won only one, and only one match (B vs N) was left.
That should have been a win for B.

N had to win two matches, so far it had won only one. So, its next win should have come from
the last empty cell (N vs S). The table so far will look like the one below.

S had already played its 1st, 2nd, and 3rd matches against Z, B, and I, respectively. Similarly, N
had already played its 2nd, 3rd, and 4th matches against I, P, and Z, respectively. So, the
match between S and N could only be their 5th match.
www.byjusexamprep.com

This concludes that the leftover match of S vs P was S’s 4th match and the leftover match of N
with B was N’s 1st match.

This further concludes that the match between I and P was their first match, the match
between I and B was their 4th match, and the match between B and P was their 5th match.
The final table looks like the one below.

Only C is true. So, option C is correct.

74. C
Sol. Let a and b be the number of sides of P1 and P2, respectively.

When then , no integer value possible

When then , no integer value possible

When then , no integer value possible

When then , no integer value possible


www.byjusexamprep.com

When then , a = 8 or a = –5 (Rejected)

When then , no integer value possible

When then , b = 7 or b= –4 (Rejected)

When then , no integer value possible

When then , no integer value possible

When then , b = 10 or b= –7

The ratio is equal to 4 : 7 when a = 8 and b = 10 (Least)

So, the required ratio is 4 : 5

Hence, option C is the correct answer.


75. C
Sol. Let T be the maximum marks and S and V be the marks obtained by Prasad and Pavan, respectively.

Then, ……(1)

Also, the following information is also given.

Passing marks =

On substituting the above value of S in (1), we get the following:


www.byjusexamprep.com

Passing percentage =

Hence, option (C) is the correct answer.

You might also like